ChaseDream

标题: Prep逻辑变体求问~ [打印本页]

作者: SW88    时间: 2017-5-27 17:59
标题: Prep逻辑变体求问~
LZ看了这题很久,发现网上这题有很多变体,实在是找不到针对这条变体的一个解释,所以上来问问nn们

74. A product that represents a cleartechnological advance over competing products can generally command a highprice. Becausetechnological advances tend to bequickly surpassed and companies want to make large profits while they stillcan, many companies charge the maximumpossible price for such a product. Butlarge profits on the new product will give competitors a strong incentiveto quickly match the new product' capabilities. Consequently, the strategy tomaximize overall profit from a new product is to charge less than thegreatest possible price.
In the argument above, the two portions in boldface play which of the followingroles?
BF1可以推出BF2,但是公司不会选择BF2
(A) The first is a consideration raised to argue that a certainstrategy is counterproductive; the second presents that strategy.
(B) The first is a consideration raised to supportthe strategy that the argument recommends; the second presents that strategy.
(C) The first is a consideration raised to helpexplain thepopularity of a certainstrategy; the second presents that strategy.
C
(D) The first is an assumption, rejected by the argument,that has been used to justify a course of action; the second presents that course of action.
(E) The first is a consideration that has been used to justifyadopting a certain strategy; the second presentsthe intended outcome of that strategy.

LZ是选了A
我的推理是这样的,BF1是可以推出BF2的,但是由于后面提到的原因,所以其实公司是不会选择BF2这个strategy的。
B选项说这个arguement recommend这种做法, 明显是错的,所以去掉
C选项这种做法其实并不popular,所以也应该去掉
D选项说BF2是一个course,所以也排除掉了
E选项说BF2是一个outcome,也被排除

那就只能选A啦,而且A不是逻辑上也很通顺么。。BF1说这种做法是事与愿违的,BF2也的确描述了这个做法。

但是正确答案是C??

晕了。。。求解答~~








作者: fxxxfxxx    时间: 2017-5-27 22:17
First is a premise to support Second, Second is a strategy to help explain the popularity of that strategy. (many companies adopt that strategy)

A is wrong because the strategy is not counterproductive. (merely the profit is less than the profit could achieved by the strategy the author proposed)
作者: SW88    时间: 2017-5-28 11:19
fxxxfxxx 发表于 2017-5-27 22:17
First is a premise to support Second, Second is a strategy to help explain the popularity of that st ...

Hi dear,

Still can't figure out how to find support of the word "popularity"..

I suppose that the sentence "Consequently, the strategy tomaximize overall profit from a new product is to charge less than thegreatest possible price." indicates no company will act like this (maximum the price).

作者: fxxxfxxx    时间: 2017-5-28 12:57
The stem states that many companies charge max price. Which is the fact in this problem.

You cannot interpret and use that interpretation as fact in CR. Not mention that interpretation violates the fact in the stem.




欢迎光临 ChaseDream (https://forum.chasedream.com/) Powered by Discuz! X3.3